GMAT Exam  >  GMAT Tests  >  Verbal for GMAT  >  Test: Critical Reasoning- 4 - GMAT MCQ

Test: Critical Reasoning- 4 - GMAT MCQ


Test Description

20 Questions MCQ Test Verbal for GMAT - Test: Critical Reasoning- 4

Test: Critical Reasoning- 4 for GMAT 2024 is part of Verbal for GMAT preparation. The Test: Critical Reasoning- 4 questions and answers have been prepared according to the GMAT exam syllabus.The Test: Critical Reasoning- 4 MCQs are made for GMAT 2024 Exam. Find important definitions, questions, notes, meanings, examples, exercises, MCQs and online tests for Test: Critical Reasoning- 4 below.
Solutions of Test: Critical Reasoning- 4 questions in English are available as part of our Verbal for GMAT for GMAT & Test: Critical Reasoning- 4 solutions in Hindi for Verbal for GMAT course. Download more important topics, notes, lectures and mock test series for GMAT Exam by signing up for free. Attempt Test: Critical Reasoning- 4 | 20 questions in 40 minutes | Mock test for GMAT preparation | Free important questions MCQ to study Verbal for GMAT for GMAT Exam | Download free PDF with solutions
Test: Critical Reasoning- 4 - Question 1

In the past, most airline companies minimized aircraft weight to minimize fuel costs. The safest airline seats were heavy, and airlines equipped their planes with few of these seats. This year the seat that has sold best to airlines has been the safest one→a clear indication that airlines are assigning a higher priority to safe seating than to minimizing fuel costs.
Q. Which of the following, if true, most seriously weakens the argument above?

Detailed Solution for Test: Critical Reasoning- 4 - Question 1

If the safest airline seats are now among the lightest, as choice E says, then buying them could be part of a strategy of minimizing fuel costs, rather than indicating a shift away from that goal. Choice E, therefore, is the best choice.
Choice A merely confirms that seat safety has improved, and thus does not weaken the argument. Many policy shifts take place without being publicly announced, so choice B does not weaken the argument. Choice C indicates that minimizing fuel costs remains a priority, but it is neutral on whether safety has become more important, so C is incorrect. Choice D does not distinguish between safe and unsafe seats, and is thus also incorrect.

Test: Critical Reasoning- 4 - Question 2

 A computer equipped with signature-recognition software, which restricts access to a computer to those people whose signatures are on file, identifies a person’s signature by analyzing not only the form of the signature but also such characteristics as pen pressure and signing speed. Even the most adept forgers cannot duplicate all of the characteristics the program analyzes.
Q. Which of the following can be logically concluded from the passage above?

Detailed Solution for Test: Critical Reasoning- 4 - Question 2

The passage asserts that skill at forging signatures is not by itself sufficient to match all of the characteristics that the software analyzes to identify signatures. Because the software gives access only after identifying a signature, access cannot be achieved by someone employing forging skill alone. Choice C is thus the best answer.
The passage gives no information about how fast the software operates or about how long the software was under development, so neither A nor D can be concluded. Choice B is incorrect since the software might have features not mentioned in the passage that make it unattractive to banks. The passages give no reason to think that errors of the sort that choice E describes, even if made, would be numerous.

1 Crore+ students have signed up on EduRev. Have you? Download the App
Test: Critical Reasoning- 4 - Question 3

Division manager: I want to replace the Microton computers in my division with Vitech computers.
General manager: Why?
Division manager: It costs 28 percent less to train new staff on the Vitech.
General manager: But that is not a good enough reason. We can simply hire only people who already know how to use the Microton computer.
Q. Which of the following, if true, most seriously undermines the general manager’s objection to the replacement of Microton computers with Vtech's?

Detailed Solution for Test: Critical Reasoning- 4 - Question 3

The general manager’s objection is based on avoiding training costs altogether. But if, as choice C says, hiring experienced users of Microton computers is significantly more costly than hiring otherwise qualified people who would have to be trained to use Vitech computers, the force of the objection is weakened. Choice C, therefore, is the best answer.
Choices A, B, and D are all incorrect; none of them provides information relevant to an evaluation of Microton computers as compared with Vitech computers. Choice E argues independently against replacing Microton computers with Vitech and thus is also incorrect.

Test: Critical Reasoning- 4 - Question 4

An airplane engine manufacturer developed a new engine model with safety features lacking in the earlier model, which was still being manufactured. During the first year that both were sold, the earlier model far outsold the new model; the manufacturer thus concluded that safety was not the customers’ primary consideration.
​Q. Which of the following, if true, would most seriously weaken the manufacturer’s conclusion?

Detailed Solution for Test: Critical Reasoning- 4 - Question 4

The manufacturers’ conclusion would be weakened if it could be argued that, in the opinion of customers, safety considerations favor the earlier model. Choice B supports such an argument and is the best answer. 
The groups mentioned in choice A would both expected to consider safety important, so their failing to buy the new model would be striking, without casting doubt on the conclusion; thus, choice A is incorrect. Choice C might support the conclusion, because customers bought other engine support the conclusion, because customers bought other engine models that might not include the newer safety features. Choice D and E suggest that usability and price, respectively, were not the customers’ primary consideration in favoring the earlier model, but neither choice weakens the conclusion that safety was not their primary consideration.

Test: Critical Reasoning- 4 - Question 5

Between 1975 and 1985, nursing-home occupancy rates averaged 87 percent of capacity, while admission rates remained constant, at an average of 95 admissions per 1,000 beds per year. Between 1985 and 1988, however, occupancy rates rose to an average of 92 percent of capacity, while admission rates declined to 81 per 1,000 beds per year.
Q. If the statements above are true, which of the following conclusions can be most properly drawn?

Detailed Solution for Test: Critical Reasoning- 4 - Question 5

Between 1985 and 1988, nursing home occupancy rates rose although admission rates declined. Choice A receives support from these facts since it would be a basis fro an adequate account of how they arose. Because it is the only choice that receives support, A is therefore the best answer.
Without information about the population of older people, nothing can be concluded about percentages in nursing homes; thus, choice B is incorrect. Since there is nothing to indicate whether the development that took place between 1985 and 1988 was an unusual development or a common one, choice C receives no support. No information about numbers of beds is provided, so neither choice D nor choice E is correct.

Test: Critical Reasoning- 4 - Question 6

Firms adopting “profit-related-pay” (PRP) contracts pay wages at levels that vary with the firm’s profits. In the metalworking industry last year, firms with PRP contracts in place showed productivity per worker on average 13 percent higher than that of their competitors who used more traditional contracts. 
Q. If, on the basis of the evidence above, it is argued that PRP contracts increase worker productivity, which of the following, if true, would most seriously weaken that argument?

Detailed Solution for Test: Critical Reasoning- 4 - Question 6

According to choice D, many firms with PRP contracts also have modernized equipment. Since the cause of their improved productivity might be the modernized equipment, not the PRP contracts, this weakens the argument, so D is the best answer.
Choice A does not weaken the argument: it is merely more evidence of the sort already being used. Choice B is incorrect because it is a natural consequence of increased worker productivity if other costs remain stable. Choice C is incorrect because it explains why introducing PRP contracts is difficult, but says nothing about the results of doing so. Choice E is incorrect because it is not implausible that workers’ pay should roughly correspond to their productivity.

Test: Critical Reasoning- 4 - Question 7

Crops can be traded on the futures market before they are harvested. If a poor corn harvest is predicted, prices of corn futures rise; if a bountiful corn harvest is predicted, prices of corn futures fall. This morning meteorologists are predicting much-needed rain for the corn-growing region starting tomorrow. Therefore, since adequate moisture is essential for the current crop’s survival, prices of corn futures will fall sharply today.
​Q. Which of the following, if true, most weakens the argument above?

Detailed Solution for Test: Critical Reasoning- 4 - Question 7

The argument, in predicting a drop in the price of corn futures, relies on news suggesting a good-sized corn crop. This prediction is undermined if there is, at the same time, news suggesting a small crop. Choice D presents such news and is therefore the best answer.
Choice A provides background information describing a stage at which rains are essential, and choice C makes rain over the entire corn-growing area seem more certain. Both are fully compatible with the argument and do nothing to weaken it. Past price changes (choice B) and details of who handles harvested corn (choice E) cannot affect the eventual size of this year’s corn crop, so neither is relevant to the argument.

Test: Critical Reasoning- 4 - Question 8

A discount retailer of basic household necessities employs thousands of people and pays most of them at the minimum wage rate. Yet following a federally mandated increase of the minimum wage rate that increased the retailer’s operating costs considerably, the retailer’s profits increased markedly.
Q. Which of the following, if true, most helps to resolve the apparent paradox?

Detailed Solution for Test: Critical Reasoning- 4 - Question 8

The question to be resolved is why the mandated wage increase, which increased operating costs, was accompanied by an increase in profits. By showing how the wage increase might have led to an increase in the retailer’s sales, choice B helps resolve this question, and thus is the best answer.
Choices A and E are incorrect, since they suggest that the wages that rose as a result of the mandated increase constituted a significant proportion of the retailer’s expenditures, which if anything adds to the seeming paradox. Choices C and D also contribute to the paradox, since they indicate that along with increases in the minimum wage there were increases in the retailer’s operating costs; so choices C and D are also incorrect.

Test: Critical Reasoning- 4 - Question 9

The cotton farms of Country Q became so productive that the market could not absorb all that they produced. Consequently, cotton prices fell. The government tried to boost cotton prices by offering farmers who took 25 percent of their cotton acreage out of production direct support payments up to a specified maximum per farm. 
Q. The government’s program, if successful, will not be a net burden on the budget. Which of the following, if true, is the best basis for an explanation of how this could be so?

Detailed Solution for Test: Critical Reasoning- 4 - Question 9

If the government’s program of support payments to cotton farmers succeeded in raising revenue for the government that would, in the absence of the program, not be raised, this could explain why the program will not be a net burden on the budget. Choice A suggests that the program would raise revenue: by raising the price of cotton, the direct support payments will boost cotton framers’ profits and thereby increase the tax revenues the government receives from cotton farmers. Therefore, A is the best answer.
None of the other choices provides a source of revenue to the government or suggests that savings would be realized in a governmental expense category, so choices B, C, D, and E are all incorrect.

Test: Critical Reasoning- 4 - Question 10

United States hospitals have traditionally relied primarily on revenues from paying patients to offset losses from unreimbursed care. Almost all paying patients now rely on governmental or private health insurance to pay hospital bills. Recently, insurers have been strictly limiting what they pay hospitals for the care of insured patients to amounts at or below actual costs.
Q. Which of the following conclusions is best supported by the information above?

Detailed Solution for Test: Critical Reasoning- 4 - Question 10

The passage explains that the primary way hospitals have covered the cost of unreimbursed care in the past is no longer available to them. It follows that they have three options: finding a new way to cover that cost, reducing it by giving less unreimbused care, or suffering a loss. This is essentially what choice B concludes, so B is the best answer.
The passage touches neither on kinds of medical procedures administered in hospitals (choice A) nor on revenue other than that received from patients or their insurers (choice E), so neither choice is correct. The passage gives no hint of who the paying patients are how do not rely on insurance, so choice C is unsupported. Concerning choice D, the passage actually suggests that it is false.

Test: Critical Reasoning- 4 - Question 11

The New Deal in America began in 1933 and included widespread bank reforms, unprecedented government infrastructure spending, and unparalleled expansion in the size of government. Some political commentators and economic historians contend that President Franklin Roosevelt's New Deal singlehandedly propelled the United States out of the Great Depression and into decades of uninterrupted prosperity. To support this claim, these economists note that during the years following 1933, GDP grew, unemployment shrunk, and optimism increased.
Q. Which of the following statements, if true, would most weaken the above argument?

Detailed Solution for Test: Critical Reasoning- 4 - Question 11

The argument's claim is: the "New Deal singlehandedly propelled the United States out of the Great Depression and into decades of uninterrupted prosperity."

The argument's evidence for this is: "during the years following 1933, GDP grew, unemployment shrunk, and optimism increased."
A. The stimulus never states that the fear sparked in "some" (note that we do not know how many) by debt spending actually exceeded the fear of not debt spending on infrastructure etc.. Further, this answer never states that the fear actually translated into a reduction in GDP or an increase in unemployment (the two main factors used to support the argument).
B. This answer merely states that World War II "expanded" economic prosperity. There is a difference between expanding prosperity and propelling a country out of a recession (i.e., the New Deal may have propelled the USA out of the depression while World War II strengthened the already growing economy). This answer does not state that World War II "propelled the United States out" of a depression and so it does not weaken the argument that it was the New Deal (not another program or event) that propelled the United States out of the Great Depression.
C. This answer speaks to the global condition while the argument pertains only to what "propelled the United States out" of the depression.
D. This answer undermines the notion that the New Deal "singlehandedly propelled the United States out of the Great Depression and into decades of uninterrupted prosperity" since the recession came back six years later.
E. This answer fails to weaken the original argument that the New Deal "singlehandedly propelled" the country out of the Great Depression since numerous other events could have propelled the country out of the depression between 1933 and 30 years after the mid-1930s.

Test: Critical Reasoning- 4 - Question 12

The strength of a suspension bridge rests in part on how deep the towers are anchored into the ground. During the first wave of suspension bridge construction, consistent with best-practices at the time, regulations required engineers to drill holes for the towers such that the portion of the tower below ground accounted for at least half of the height of the tower. After conducting an inspection into the depth of the holes drilled for the towers of the Watergate Bridge, constructed over 50 years ago during the first wave of suspension bridge construction, regulators noted that updated architectural norms and theory advised that the bridge's towers should be reinforced to meet anticipated increases in usage.
Q. Which of the following is most strongly supported by the information above?

Detailed Solution for Test: Critical Reasoning- 4 - Question 12

The conclusion of the stimulus is: "regulators noted that updated architectural norms and theory advised that the bridge's towers should be reinforced to meet anticipated increases in usage".

A. Since the bridge "should be reinforced to meet anticipated increases in usage," the author implies that the bridge is currently safe but should be reinforced to prepare for future changes in usage.
B. Two reasons are given for reinforcing: (1) "updated architectural norms" (2) "to meet anticipated increases in usage." Since the changes must be made for these two reasons, we can conclude that the original standards did not anticipate these two reasons/factors.
C. This answer states that the bridge still "will not be safe." The passage never states or implies that the bridge will not be safe. In fact, the words "to meet" seem to imply that the bridge will be safe with the changes.
D. The new architectural norms advocate the addition of supplemental reinforcements for the Watergate Bridge "to meet anticipated increases in demand." We cannot conclude that the historic regulations are faulty or that the situation with the Watergate Bridge mirrors that of every other bridge (i.e., since not every bridge is like the Watergate in its "anticipated increases in usage," we cannot conclude that every bridge needs the same reinforcements as the Watergate does).
E. The action should be undertaken because it is advised by regulators and is consistent with current design theory. The bridge was originally constructed "consistent with best-practices at the time" and the new reinforcements will help "meet anticipated increases in usage." The stimulus gives no indication that current best-practices should be abandoned simply because it is impossible to predict future theory.

Test: Critical Reasoning- 4 - Question 13

Automation, the trademark of a modern economy, is essential to maximizing a country's economic production while minimizing its costs. Health care executives want to increase revenues while reducing costs. Consequently, they propose significantly greater automation of health care. Yet, this should be rejected. Radical automation of health care would cause patients to lose trust in the system as the health care they would receive would lack the in-person care that studies show patients desire.
Q. Which of the following expresses the main point of the argument?

Test: Critical Reasoning- 4 - Question 14

For years, a considerable number of students on West County High School's track team complained about shin splints (medial tibial syndrome). However, during the most recent season, the number of students who complained about shin splints dropped significantly. School officials assert that this reduction in complaints occurred entirely as a result of the school's decision to build a new running track that provided a softer running surface, which absorbed much of the shock on the knees and shins that occurs when running and causes shin splints.
Q. Which of the following, if true, most severely weakens the school officials' explanation for the decrease in complaints about shin splints?

Detailed Solution for Test: Critical Reasoning- 4 - Question 14

The school authorities argue that the new track "entirely" caused the reduction in "complaints" about shin splints. There are two important points about this argument.

(1) The argument of the school administrators is based upon a reduction in the number of complaints about shin splints, which is not the same as a reduction in the number of actual instances. It is entirely possible that students complain about shin splints and yet actually have other problems.

(2) The argument of the school administrators established a causal relationship (i.e., the new track caused the reduction). This is a much more assertive and broad claim than simply noticing that the two are correlated (i.e., occurred together).

A. The argument made by West County High School officials is based upon the fact that "the number of students who complained about shin splints dropped significantly." This answer would weaken an argument that dealt with the number of diagnosed instances of shin splints. However, the school administrators make their argument only because of a reduction in the number of "claims."
B. This answer significantly strengthens the argument of the high school administrators by noting that other schools experienced a link between a new track and a decrease in claims about shin splints.
C. This answer calls into question the school's assertion that the new track was "entirely" responsible for the reduction in claims of shin splints. The answer does this through providing an alternative and viable (but not necessarily competing) explanation of the reduction in claims of shin splints.
D. The original argument pertains to complaints about shin splints in particular (not the number of students who "complained of pain while running"). This answer confuses complaints in general with complaints about shin splints in particular.
E. This answer strengthens the school officials' claim instead of weakening it as it provides more evidence that the new track helped decrease complaints of shin splints.

Test: Critical Reasoning- 4 - Question 15

After thousands of miles of use, the tread on many bike tires wears down. One common theory about why tires wear down contends that the perpetual friction and heat generated by the contact between the tire and pavement erode the material on the surface of the tire. However, a local scientist who is also an avid cyclist proposed a new theory for why bike tires wear down. This scientist contended that chemicals from the road's composition and chemicals from rain residue wore down the surface of the tire.
Q. Which of the following would best evaluate the veracity of the scientist's proposed theory?

Detailed Solution for Test: Critical Reasoning- 4 - Question 15

There are two different proposed causes of tire decay on bikes: (1) "contact between the tire and pavement" (2) "chemicals from the road's composition and chemicals from rain residue"

In order to ascertain which theory is correct, we must separate the two proposed causes. This can be done by applying chemicals from rain water and pavement (theory 2) onto a bike's idle tires. It is important that the bike is idle as this rules out theory 1.

A. This method does not isolate the new and proposed theory for why bike tires wear down as the testing method exposes the tires to both potential causes (i.e.: (1) tire and pavement contact (2) chemicals from the road's composition and rain residue).
B. This method isolates the two competing sources of explanations for the tires' erosion. If the tires erode under this scenario, we know that it was not riding on the pavement that caused the erosion but rather it was the exposure to chemicals.
C. This method does not account for both the potential influence of chemicals from the road's composition and chemicals from rain residue on the surface of the bike tire. Simply because the manufacturer used or did not use chemicals from the road's composition within the bike's frame does not enable us to conclude that the same chemicals will or will not have a corrosive effect if applied to the bike's tires. In other words, testing effects on the bike's frame is entirely unrelated to testing effects on the bike's tires as these two components (i.e., frame and tire) are made of entirely different materials.
D. This method does not account for the potential influence of chemicals from the road's composition and chemicals from rain residue on the surface of the bike tire. Simply because the manufacturer used or did not use rust-resistant components for the bike's frame does not enable us to conclude what effect water may have on the bike's tires. In other words, testing effects on the bike's frame is entirely unrelated to testing effects on the bike's tires as these two components are made of entirely different materials.
E. Without any useful frame of reference (such as the number of miles a tire can be used on a dry road or the number of miles a tire can be used in a grass field without the chemicals of a typical road), it is impossible to make any logical conclusion about whether friction or chemicals are causing the decay of the tire.

Test: Critical Reasoning- 4 - Question 16

During the past 20 years, computer scientists focused increasingly on starting and running successful businesses. However, since businesses must be profitable, computer scientists must focus on developing products that generate profit. Consequently, computer science has lost its creative aspect.
Q. Which of the following assumptions is most necessary in order for the conclusion above to be drawn from the argument above?

Detailed Solution for Test: Critical Reasoning- 4 - Question 16

The argument’s conclusion is: "computer science has lost its creative aspect"

The argument draws the false dichotomy that the writing of profitable programs requires forgoing the writing of creative applications. In other words, it assumes that profitable and creative programs are mutually exclusive. Further, it assumes that all computer scientists are pursuing writing profitable programs.

A. There is a difference between being well received and being profitable. The argument assumes that only non-creative programs can be profitable. However, this does not mean creative programs will not be well-received. It just means they will not be profitable.
B. Even if some computer scientists disregarded creativity, we cannot (as the original argument and this answer do) assume that creativity and profit are mutually exclusive.
C. This answer identifies a crucial assumption in the original argument. If writing creative and profitable programs were not mutually exclusive, then one could write profitable programs without "computer science [having] lost its creative aspect."
D. The extent to which a computer scientist is obsessed with the profitability of his work has no influence on whether profitability drives away creativity (as the original argument assumes).
E. The users of software have influence on whether software can be both profitable and creative.

Test: Critical Reasoning- 4 - Question 17

Lauren is clearly going to make an awful professor. Nearly half of her students failed their final this past spring. She should probably choose another career path, because her students’ performance demonstrates that she doesn’t teach very well.

What statement, if true, most weakens the argument above?

Detailed Solution for Test: Critical Reasoning- 4 - Question 17

C is correct because this shows that the students' poor attendance, and not Lauren's teaching style, was likely to blame for their poor performance on the exam. If they were not in class when Lauren was teaching, her style could have very little positive or negative impact on their learning.

A is incorrect because we do not know if Lauren's students did better, worse, or the same in the previous class than in this one.

B is incorrect because the class being required would have no direct impact on how students would do on a single exam.

D is incorrect because it speaks to the motivation of the students, and perhaps the teaching effectiveness of the TA, not Lauren herself.

E is incorrect because it supports that idea that Lauren has had issues with teaching effectiveness.

Test: Critical Reasoning- 4 - Question 18

Three years ago, Ron gave up eating anything with wheat in it. In that time, he lowered his cholesterol by 100 points and lost nearly 85 pounds. He says he feels better than he has in years. As a result, he is writing a book which says that everyone should stop eating wheat products. He says people are not made to eat wheat and that giving it up is the key to losing weight and being healthier.

Which of the following, if true, most weakens the argument above?

Detailed Solution for Test: Critical Reasoning- 4 - Question 18

B is correct because Ron's family history means he may be intolerant of wheat. Thus quitting wheat would improve his health, but this might not hold true for people without this sensitivity.

A is incorrect because the presence or absence of wheat in any given food does not effect the validity of Ron's argument.

C is incorrect because low carbohydrate diets do not necessarily entail giving up wheat; even those that do would actually bolster Ron's argument.

D is incorrect because Ron's running likely helped his health improve, but does not negate the impact giving up wheat had on his health. His argument is concerned with wheat, not exercise.

E is incorrect because Ron is not making an assertion about the benefits or drawbacks of either a vegetarian or meat-based diet.

Test: Critical Reasoning- 4 - Question 19

Gina doesn't understand why she cannot get a job as a computer engineer. She even went back to school and got a degree in computer science. After two years on the job market with only a few interviews and no offers, she is staring to wonder if she is not getting hired because she is a woman. One of her friends told her that women seldom succeed in technology fields, and she is beginning to believe it.

Which of the following, if true, most weakens the argument above?

Detailed Solution for Test: Critical Reasoning- 4 - Question 19

D is correct because it indicates that Gina's grades and knowledge of the field, not her sex, are likely behind her failure to get hired.

A is incorrect because whether her degree was earned online or not likely matters little; hiring managers probably don't even know whether her degree was earned online.

B is incorrect because, while it indicates that there are many men in the field, Gina has had only a few interviews. These experiences do not mean that women are not hired.

C is incorrect because it indicates that both men and women teach in the field, thus that women get hired.

D is incorrect because while she might interview well, this does not guarantee an offer when weighed against other factors on her resume.

Test: Critical Reasoning- 4 - Question 20

It seems like every day you turn on the local news and you hear about someone being murdered in our city. It is really frightening. I am not sure I want to live somewhere so violent, and crime is clearly going up here. Otherwise the news shows wouldn't have so many murders to report.

Which of the following, if true, most weakens the argument above?

Detailed Solution for Test: Critical Reasoning- 4 - Question 20

C is correct because the speaker is basing his/her conclusion on the amount of news coverage of murders, Since the station may be increasing coverage even if crimes rates are falling, the impression that more news equates with more crime in not necessarily true.

A is incorrect because it does not speak to the speaker's perception that more news equates to more crime.

B is incorrect because it is unknown whether the media has responded to the groups' urging to cover these crimes, nor is it known if these crimes are on the increase or decrease.

D is incorrect because, while watching multiple news shows a day might give one a perception of more coverage, it does not speak to whether the actual crime rate is rising.

E is incorrect because it does not speak to the speaker's perception of more crime.

52 videos|54 docs|61 tests
Information about Test: Critical Reasoning- 4 Page
In this test you can find the Exam questions for Test: Critical Reasoning- 4 solved & explained in the simplest way possible. Besides giving Questions and answers for Test: Critical Reasoning- 4, EduRev gives you an ample number of Online tests for practice

Top Courses for GMAT

52 videos|54 docs|61 tests
Download as PDF

Top Courses for GMAT